-
Notifications
You must be signed in to change notification settings - Fork 2
/
lista16.tex
83 lines (79 loc) · 4.34 KB
/
lista16.tex
1
2
3
4
5
6
7
8
9
10
11
12
13
14
15
16
17
18
19
20
21
22
23
24
25
26
27
28
29
30
31
32
33
34
35
36
37
38
39
40
41
42
43
44
45
46
47
48
49
50
51
52
53
54
55
56
57
58
59
60
61
62
63
64
65
66
67
68
69
70
71
72
73
74
75
76
77
78
79
80
81
82
83
% Filename: lista16.tex
%
% This code is part of 'Solutions for MT402, Matrizes'
%
% Description: This file corresponds to the solutions of homework sheet 16.
%
% Created: 23.06.12 12:21:04 PM
% Last Change: 29.06.12 05:35:26 PM
%
% Authors:
% - Raniere Silva (2012): initial version
%
% Copyright (c) 2012 Raniere Silva <[email protected]>
%
% This work is licensed under the Creative Commons Attribution-ShareAlike 3.0 Unported License. To view a copy of this license, visit http://creativecommons.org/licenses/by-sa/3.0/ or send a letter to Creative Commons, 444 Castro Street, Suite 900, Mountain View, California, 94041, USA.
%
% This work is distributed in the hope that it will be useful, but WITHOUT ANY WARRANTY; without even the implied warranty of MERCHANTABILITY or FITNESS FOR A PARTICULAR PURPOSE.
%
Antes de mais nada \'{e} importante um pequeno exclarecimento:
\begin{quote}
A partir de agora \'{e} importante fazer a distin\c{c}\~{a}o entre a decomposi\c{c}\~{a}o QR e o algoritmo/m\'{e}todo QR. O algorimo QR \'{e} um procedimento iterativo para encontrar autovalores. Ele \'{e} baseado na decomposi\c{c}\~{a}o QR, que \'{e} um procedimento direto relacionado com o procedimento de Gram-Schmidt. (Tradu\c{c}\~{a}o livre de trecho de ``Fundamentals of Matrix Computations''\nocite{Watkins:2004:fundamentals}, p\'{a}gina 357)
\end{quote}
\begin{questions}
\question Sejam $H$ uma matriz Hessenberg superior\footnote{Todos os elementos abaixo da primeira subdiagonal s\~{a}o zeros.} e $R$ triangular superior. Mostre que $H R$ e $R H$ tamb\'{e}m s\~{a}o Hessenberg superior.
\begin{solution}
% TODO Fazer esse exerc\'{i}cio.
\end{solution}
\question Considere a matriz n\~{a}o singular $A$ particionada da seguinte forma:
\begin{align*}
A = \begin{bmatrix}
A_{11} & A_{12} \\
0 & A_{22}
\end{bmatrix},
\end{align*}
onde $A_{11}$ e $A_{22}$ s\~{a}o matrizes quadradas. Seja $A = Q R$ e considere a matriz ortogonal $Q$ e a matriz triangular superior $R$ particionadas de forma equivalente. Mostre que $Q_{12} = Q_{21} = 0$ e que $Q_{11}$ s\~{a}o ortogonais. Mostre tamb\'{e}m que $A_{11} = Q_{11} R_{11}$ e que $A_{22} = Q_{22} R_{22}$.
\begin{solution}
% TODO Fazer esse exerc\'{i}cio.
\end{solution}
\question Considere uma matriz $A : 2 \times 2$. Mostre que os autovalores de $\sigma_0$ e $\sigma_1$ desta matriz satisfazem $\sigma_0 + \sigma_1 = \mathrm{trace}(A)$ e $\sigma_0 \sigma_1 = \det(A)$.
\begin{solution}
Consideremos que
\begin{align*}
A &= \begin{bmatrix}
a_{11} & a_{12} \\
a_{21} & a_{22}
\end{bmatrix}.
\end{align*}
Ent\~{a}o os autovalores de $A$ correspondem aos valores de $\sigma$ tal que
\begin{align*}
(A - \sigma I) x = 0 \Rightarrow \det(A - \sigma I) = 0.
\end{align*}
Como $A$ \'{e} uma matriz dois por dois sabemos calcular o determinante de $A$, que \'{e}
\begin{align*}
\det(A) &= a_{11} a_{22} - a_{12} a_{21},
\end{align*}
e tamb\'{e}m de $(A - \sigma I)$, que corresponde a
\begin{align*}
\det(A - \sigma I) &= (a_{11} - \sigma) (a_{22} - \sigma) - a_{12} a_{21} \\
&= \sigma^2 - (a_{11} + a_{22}) \sigma + a_{11} a_{22} - a_{12} a_{21}.
\end{align*}
Como $\det(A - \sigma I) = 0$ e pela express\~{a}o acima que \'{e} uma equa\c{c}\~{a}o temos que
\begin{align*}
\sigma_1 + \sigma_2 &= a_{11} + a_{22} = \mathrm{trace}(A), \\
\sigma_1 \sigma_2 &= a_{11} a_{22} - a_{12} a_{21} = \det(A).
\end{align*}
\end{solution}
\question Seja $H$ uma matriz Hessenberg superior cujos autovalores s\~{a}o conhecidos. Compare o esfor\c{c}o computacional para calcular os autovetores de $H$ pelo m\'{e}todo das pot\^{e}ncias e pelo m\'{e}todo QR.
\begin{solution}
% TODO Fazer esse exerc\'{i}cio.
\end{solution}
\question Considere o m\'{e}todo QR com \textit{shift} $\sigma^k$ aplicado \`{a} matriz $H^k$ na itera\c{c}\~{a}o $k$. Mostre que
\begin{align*}
\prod_{k = 1}^j Q^k \prod_{k = j}^1 R^k = \prod_{k = 1}^j \left( H - \sigma^k I \right),
\end{align*}
onde $H = H^0$ \'{e} a matriz original.
\begin{solution}
% TODO Fazer esse exerc\'{i}cio.
\end{solution}
\end{questions}